Difference between revisions of "2022 AMC 10A Problems/Problem 1"

(Created page with "==Problem== What is the value of <cmath>3+\frac{1}{3+\frac{1}{3+\frac13}}?</cmath> <math>\textbf{(A)}\ \frac{31}{10}\qquad\textbf{(B)}\ \frac{49}{15}\qquad\textbf{(C)}\ \frac{...")
 
(redirection)
Line 1: Line 1:
 +
{{duplicate|[[2022 AMC 10A Problems/Problem 1|2022 AMC 10A #1]] and [[2022 AMC 12A Problems/Problem 1|2022 AMC 12A #1]]}}
 +
 
==Problem==
 
==Problem==
 
What is the value of <cmath>3+\frac{1}{3+\frac{1}{3+\frac13}}?</cmath>
 
What is the value of <cmath>3+\frac{1}{3+\frac{1}{3+\frac13}}?</cmath>
Line 17: Line 19:
  
 
{{AMC10 box|year=2022|ab=A|before=First Problem|num-a=2}}
 
{{AMC10 box|year=2022|ab=A|before=First Problem|num-a=2}}
 +
{{AMC12 box|year=2022|ab=A|before=First Problem|num-a=2}}
 
{{MAA Notice}}
 
{{MAA Notice}}

Revision as of 20:50, 11 November 2022

The following problem is from both the 2022 AMC 10A #1 and 2022 AMC 12A #1, so both problems redirect to this page.

Problem

What is the value of \[3+\frac{1}{3+\frac{1}{3+\frac13}}?\] $\textbf{(A)}\ \frac{31}{10}\qquad\textbf{(B)}\ \frac{49}{15}\qquad\textbf{(C)}\ \frac{33}{10}\qquad\textbf{(D)}\ \frac{109}{33}\qquad\textbf{(E)}\ \frac{15}{4}$

Solution

We have \begin{align*} 3+\frac{1}{3+\frac{1}{3+\frac13}} &= 3+\frac{1}{3+\frac{1}{\left(\frac{10}{3}\right)}} \\ &= 3+\frac{1}{3+\frac{3}{10}} \\ &= 3+\frac{1}{\left(\frac{33}{10}\right)} \\ &= 3+\frac{10}{33} \\ &= \boxed{\textbf{(D)}\ \frac{109}{33}}. \end{align*} ~MRENTHUSIASM

See Also

2022 AMC 10A (ProblemsAnswer KeyResources)
Preceded by
First Problem
Followed by
Problem 2
1 2 3 4 5 6 7 8 9 10 11 12 13 14 15 16 17 18 19 20 21 22 23 24 25
All AMC 10 Problems and Solutions
2022 AMC 12A (ProblemsAnswer KeyResources)
Preceded by
First Problem
Followed by
Problem 2
1 2 3 4 5 6 7 8 9 10 11 12 13 14 15 16 17 18 19 20 21 22 23 24 25
All AMC 12 Problems and Solutions

The problems on this page are copyrighted by the Mathematical Association of America's American Mathematics Competitions. AMC logo.png